Đến nội dung

Nguyen Minh Hai nội dung

Có 652 mục bởi Nguyen Minh Hai (Tìm giới hạn từ 10-05-2020)



Sắp theo                Sắp xếp  

#567121 Topic tổng hợp một số bất đẳng thức trong kì thi MO các nước

Đã gửi bởi Nguyen Minh Hai on 20-06-2015 - 16:19 trong Bất đẳng thức - Cực trị

Bài 128(Mở rộng khác của bài 118)(tự sáng tác): Cho a,b,c>0. CMR:

$\sqrt[4]{\frac{a^4+b^2+1}{21(b^2+c^2)+6ca}}+\sqrt[4]{\frac{b^4+c^2+1}{21(c^2+a^2)+6ca}}+\sqrt[4]{\frac{c^4+a^2+1}{21(a^2+b^2)+6ab}}\geq \frac{3}{2}$

Hướng giải là tương tự bài 118

$AM-GM$ trực tiếp cho 3 số hạng vế trái và chú ý

$\prod(a^4+b^2+1)\geq(a^2+b^2+c^2)^3$

Còn cái dưới mẫu sau khi sử dụng $AM-GM$ ta tiếp tục đánh giá bằng $AM-GM$




#567114 Topic tổng hợp một số bất đẳng thức trong kì thi MO các nước

Đã gửi bởi Nguyen Minh Hai on 20-06-2015 - 15:56 trong Bất đẳng thức - Cực trị

Bài 125:(Guzia MO) : Cho a,b,c >0: a+b+c=3. CMR:

 

$\sqrt[7]{\frac{a^3+2}{b(c+2a)}}+\sqrt[7]{\frac{b^3+2}{c(a+2b)}}+\sqrt[7]{\frac{c^3+2}{a(b+2c)}}\geq \frac{9abc}{a^2+b^2+c^2}$

Áp dụng $AM-GM$ cho vế trái ta được:

$VT \geq 3\sqrt[21]{\frac{(a^3+2)(b^3+2)(c^3+2)}{abc(a+2b)(b+2c)(c+2a)}}$                                    $(1)$

 

Theo BĐT $Holder$ ta lại có: $(a^3+2)(b^3+2)(c^3+2)\geq(a+b+c)^3=27$                                        $(2)$

 

Lại áp dụng BĐT $AM-GM$ ta được:

$abc(a+2b)(b+2c)(c+2a)\leq\left ( \frac{a+b+c}{3} \right )^3.\left ( \frac{3a+3b+3c}{3} \right )^3=27$  $(3)$

 

Từ $(1)$, $(2)$ và $(3)$ suy ra $VT \geq 3$

 

Mặt khác ta có: $a^2+b^2+c^2=\frac{1}{3}(a+b+c)(a^2+b^2+c^2)\geq 3abc$

$\Rightarrow VP=\frac{9abc}{a^2+b^2+c^2}\leq 3$

 

Do đó BĐT được chứng minh! 




#567103 Topic tổng hợp một số bất đẳng thức trong kì thi MO các nước

Đã gửi bởi Nguyen Minh Hai on 20-06-2015 - 15:37 trong Bất đẳng thức - Cực trị

Bài 123:(Mở rộng của bài 118)(tự sáng tác) : Cho a,b,c>0. Tìm hằng số k tốt nhất sao cho bđt sau đúng:

 

$\sqrt{\frac{a^3+b^3+1}{k(b^2+c^2)+13bc}}+\sqrt{\frac{b^3+c^3+1}{k(c^2+a^2)+13ca}}+\sqrt{\frac{c^3+a^3+1}{k(a^2+b^2)+13ab}}\geq 1$

Sử dụng BĐT $AM-GM$ cho vế trái ta được:

$\sum \sqrt{\frac{a^3+b^3+1}{k(b^2+c^2)+13bc}} \geq 3.\sqrt[6]{\frac{\prod( a^3+b^3+1)}{\prod(k(b^2+c^2)+13bc) }}$

 

Theo BĐT $Holder$ ta lại có:

$\prod (a^3+b^3+1)\geq(a^2+b^2+1)^3$

 

Và theo $AM-GM$ ta có:

$\prod \left [ k(b^2+c^2)+13bc \right ]\leq\prod \left ( k+\frac{13}{2} \right )(b^2+c^2)$

$\leq \frac{8}{27}.\left ( k+\frac{13}{2} \right )^3(a^2+b^2+c^2)^3$

 

Do đó: $VT \geq 3.\sqrt[6]{\frac{27}{8(k+\frac{13}{2})^3}}=3\sqrt{\frac{3}{2(k+\frac{13}{2})}} \geq 1$

$\Rightarrow k \leq 7$

 

Do đó hằng số $k$ tốt nhất để BĐT đề bài đùng là $k=7$




#567606 Topic tổng hợp một số bất đẳng thức trong kì thi MO các nước

Đã gửi bởi Nguyen Minh Hai on 23-06-2015 - 10:55 trong Bất đẳng thức - Cực trị

Không biết bài này có chưa, nếu có rồi thì anh Đức xóa dùm em

Bài 142(APMO 1996) Cho a,b,c là độ dài 3 cạnh tam giác. Chứng minh rằng:

$\sqrt{a+b-c}+\sqrt{b+c-}+\sqrt{c+a-b}\geq \sqrt{a}+\sqrt{b}+\sqrt{c}$

Đẳng thức xảy ra khi nào

Đề sai rồi. Đề đúng phải là: $\sqrt{a+b-c}+\sqrt{b+c-a}+\sqrt{c+a-b} \leq \sqrt{a}+\sqrt{b}+\sqrt{c}$

Ta đặt: $\left\{\begin{matrix} a+b-c=z & & & \\ b+c-z=x & & & \\ c+a-b =y& & & \end{matrix}\right.$

$\Rightarrow a=\frac{y+z}{2};b=\frac{z+x}{2};c=\frac{x+y}{2}$

 

BĐT cần chứng minh trở thành: $\sqrt{2}(\sqrt{x}+\sqrt{y}+\sqrt{z}) \leq \sqrt{x+y}+\sqrt{y+z}+\sqrt{z+x}$

$\Leftrightarrow 2(\sqrt{x}+\sqrt{y}+\sqrt{z}) \leq \sqrt{2(x+y)}+\sqrt{2(y+z)}+\sqrt{2(x+z)}$

 

Áp dụng BĐT $Buniakowski$ ta có:

$\left\{\begin{matrix} \sqrt{2(x+y)} \geq \sqrt{x}+\sqrt{y} & & & \\ \sqrt{2(y+z)} \geq \sqrt{y}+\sqrt{z}& & & \\ \sqrt{2(z+x)} \geq \sqrt{z}+\sqrt{x} & & & \end{matrix}\right.$

 

Do đó ta có đpcm.




#567722 Topic tổng hợp một số bất đẳng thức trong kì thi MO các nước

Đã gửi bởi Nguyen Minh Hai on 23-06-2015 - 20:40 trong Bất đẳng thức - Cực trị

Bài 144(Poland MO): Cho a,b,c>0. CMR: nếu ab+bc+ca=1 thì

$\frac{(a^2+1)^2}{bc(b+c)}+\frac{(b^2+1)^2}{ca(c+a)}+\frac{(c^2+1)^2}{ab(a+b)}\geq 8(\sqrt{ab}+\sqrt{bc}+\sqrt{ca})$

Bổ đề 1: $a)$   $x+y+z \geq \sqrt{3(xy+yz+zx)}=\sqrt{3}$

               $b)$   $ 3xyz(x+y+z) \leq (xy+yz+zx)^2$                    

                     (2 BĐT này quen thuộc rồi)

 

Bồ đề 2: $(x^2+3)(y^2+3)(z^2+3) \geq 4(x+y+z+1)^2$

[spoiler] Chứng minh:

Không mất tính tổng quát, giả sử: $(x^2-1)(y^2-1) \geq 0$

 

Khi đó:    $(x^2+3)(y^2+3)=(x^2-1)(y^2-1) + 4(x^2+y^2+2) \geq 4(x^2+y^2+2)$\

 

Nên ta cần chứng minh:

$(x^2+y^2+2)(z^2+3) \geq (x+y+z+1)^2$

 đúng theo BĐT $Cauchy-Schwarz:$

$(x^2+y^2+1+1)(1+1+z^2+1) \geq (x+y+z+1)^2$

Xảy ra dấu $"="$ khi $x=y=z=1$\

[\spoiler]

 

Ta đi vào bài toán, áp dụng BĐT $AM-GM$ cho vế trái ta được:

$\sum \frac{(a^2+1)^2}{bc(b+c)}\geq 3\sqrt[3]{\frac{(a^2+1)^2(b^2+1)^2(c^2+1)^2}{a^2b^2c^2(a+b)(b+c)(c+a)}}$

 

Áp dụng BĐT $AM-GM$ ta có: 

  $a^2b^2c^2(a+b)(b+c)(c+a) \leq \frac{8}{27}a^2b^2c^2(a+b+c)^3$

$=\frac{8}{3^5}(a+b+c).9a^2b^2c^2(a+b+c)^2$

$\leq \frac{8}{3^5}.(a+b+c).(ab+bc+ca)^4=\frac{8}{3^5}(a+b+c)$                    

 

Áp dụng Bổ đề 2 ta có:

$\prod (a^2+1)=\frac{1}{27}.\prod \left [ (a\sqrt{3})^2+3 \right ]$

$\geq \frac{4}{27}.\left [ (a+b+c)\sqrt{3}+1 \right ]^2$

 

Do đó 

$\frac{(a^2+1)^2(b^2+1)^2(c^2+1)^2}{a^2b^2c^2(a+b)(b+c)(c+a)} \geq \frac{2}{3}. \frac{\left [ (a+b+c)\sqrt{3}+1 \right ]^4}{a+b+c}$

 

$=\frac{2}{3}\left [ \frac{3(a+b+c)^2+2\sqrt{3}(a+b+c)+1}{a+b+c} \right ].\left [ (a+b+c)\sqrt{3}+1 \right ]^2$

 

$=\frac{2}{3}\left [ \frac{8}{3}(a+b+c)+\left ( \frac{1}{3}(a+b+c)+\frac{1}{a+b+c} \right )+2\sqrt{3} \right ].\left [ (a+b+c)\sqrt{3}+1 \right ]$

 

$\geq \frac{2}{3}\left [ \frac{8}{3}.\sqrt{3}+\frac{2}{\sqrt{3}}+2\sqrt{3} \right ].[\sqrt{3}.\sqrt{3}+1]^2=\frac{512\sqrt{3}}{9}$

 

$\Rightarrow VT \geq 3\sqrt[3]{\frac{512\sqrt{3}}{9}}=8\sqrt{3}$

 

Mặt khác: $VP=8(\sqrt{ab}+\sqrt{bc}+\sqrt{ca})\leq 8.\sqrt{3(ab+bc+ca)}=8\sqrt{3}$

 

Do đó bài toán được chứng minh! Xảy ra dấu $"="$ khi $a=b=c=\frac{1}{\sqrt{3}}$




#568182 Topic tổng hợp một số bất đẳng thức trong kì thi MO các nước

Đã gửi bởi Nguyen Minh Hai on 25-06-2015 - 21:41 trong Bất đẳng thức - Cực trị

Bài 153: (Việt Nam TST 2005) 

Cho $a,b,c>0$. Chứng minh: $\left ( \frac{a}{a+b} \right )^3+\left ( \frac{b}{b+c} \right )^3+\left ( \frac{c}{c+a} \right )^3 \geq \frac{3}{8}$




#567739 Topic tổng hợp một số bất đẳng thức trong kì thi MO các nước

Đã gửi bởi Nguyen Minh Hai on 23-06-2015 - 21:44 trong Bất đẳng thức - Cực trị

Bài 146: (Vietnamese IMO Team Selection Test 1993)

Cho các số thực $a,b,c,d$ thõa mãn: $\frac{1}{2} \leq a^2+b^2+c^2+d^2 \leq 1$

Tìm giá trị lớn nhất và giá trị nhỏ nhất của biểu thức:

$P=(a-2b+c)^2+(b-2c+d)^2+(b-2a)^2+(c-2d)^2$




#566962 Topic tổng hợp một số bất đẳng thức trong kì thi MO các nước

Đã gửi bởi Nguyen Minh Hai on 19-06-2015 - 21:01 trong Bất đẳng thức - Cực trị

 

Bài 116 (CĐTMO 2005) : Chứng minh rằng
                                                                 $\frac{a^{3}}{(b+c)^{3}}+\frac{b^{3}}{(c+a)^{3}}+\frac{c^{3}}{(a+b)^{3}}$ $\geq \frac{3}{8}$

                                        trong đó $a,b,c$ là các số dương.

 

 

Ta có BĐT quen thuộc: $\frac{x^3+y^3+z^3}{3}\geq \left ( \frac{x+y+z}{3} \right )^3$ (Chứng minh bằng $AM-GM$)

Áp dụng BĐT trên ta được:

$VT \geq \frac{1}{9}\left (\frac{a}{b+c}+\frac{b}{c+a}+\frac{c}{a+b} \right )^3\geq \frac{1}{9}.\frac{27}{8}=\frac{3}{8}$




#567548 Topic tổng hợp một số bất đẳng thức trong kì thi MO các nước

Đã gửi bởi Nguyen Minh Hai on 22-06-2015 - 22:41 trong Bất đẳng thức - Cực trị

Bài 139(Kazakstan MO): Cho a,b,c>0. CMR:

$\sum \frac{1}{a^2+ab+b^2}\geq \frac{9}{(a+b+c)^2}$

BĐT cần chứng minh tương đương với

$\sum \frac{a^2+b^2+c^2+ab+bc+ca}{a^2+ab+b^2} \geq \frac{9(a^2+b^2+c^2+ab+bc+ca)}{(a+b+c)^2}$

 

$\Leftrightarrow \sum \left ( \frac{(a^2+ab+b^2)+c(a+b+c)}{a^2+b^2+c^2} \right ) \geq \frac{9(a^2+b^2+c^2+ab+bc+ca)}{(a+b+c)^2}$

 

$\Leftrightarrow 3+(a+b+c).\sum \frac{c}{a^2+ab+b^2} \geq \frac{9(a^2+b^2+c^2+ab+bc+ca)}{(a+b+c)^2}$

 

Áp dụng $Cauchy-Schwarz$ ta có:

$\sum \frac{c}{a^2+ab+b^2}\geq \frac{(a+b+c)^2}{\sum c(a^2+ab+b^2)}=\frac{a+b+c}{ab+bc+ca}$

 

Do đó ta cần chứng minh:

$3+\frac{(a+b+c)^2}{ab+bc+ca} \geq \frac{9(a^2+b^2+c^2+ab+bc+ca)}{(a+b+c)^2}$

 

$\Leftrightarrow 3+\frac{(a+b+c)^2}{ab+bc+ca}\geq \frac{9[(a+b+c)^2-(ab+bc+ca)]}{(a+b+c)^2}$

 

$\Leftrightarrow \frac{(a+b+c)^2}{ab+bc+ca}+\frac{9(ab+bc+ca)}{(a+b+c)^2} \geq 6$

 

Tuy nhiên BĐT cuối luôn đúng theo $AM-GM$

Do đó bài toán được chứng minh xong!




#566811 Topic tổng hợp một số bất đẳng thức trong kì thi MO các nước

Đã gửi bởi Nguyen Minh Hai on 19-06-2015 - 09:48 trong Bất đẳng thức - Cực trị

 

Bài 116 (CĐTMO 2005) : Chứng minh rằng
                                                                 $\frac{a^{3}}{(b+c)^{3}}+\frac{b^{3}}{(c+a)^{3}}+\frac{c^{3}}{(a+b)^{3}}$

                                        trong đó $a,b,c$ là các số dương.

 

 

Có vẽ đề bị thiếu thì phải?  :closedeyes:




#566654 Topic tổng hợp một số bất đẳng thức trong kì thi MO các nước

Đã gửi bởi Nguyen Minh Hai on 18-06-2015 - 16:32 trong Bất đẳng thức - Cực trị

Bài 111(Greece MO)(Nguyễn Duy Khương): Cho a,b,c là các số thực dương sao cho ab+bc+ca=1 . Cmr:

 

                           

                          $\sqrt{\frac{a^2+1}{b^2+6bc+c^2}}+\sqrt{\frac{b^2+1}{c^2+6ca+a^2}}+\sqrt{\frac{c^2+1}{a^2+6ab+b^2}}\geq \frac{3\sqrt{2}}{2}$

Áp dụng BĐT $AM-GM$ ta có:

$\sum \sqrt{\frac{a^2+1}{b^2+6bc+c^2}}=\sum \sqrt{\frac{a^2+ab+bc+ca}{(b+c)^2+4bc}}$

 

$=\sum \sqrt{\frac{(a+b)(c+a)}{(b+c)^2+4bc}}$

 

$ \geq \sum \sqrt{\frac{(a+b)(c+a)}{2(b+c)^2}}$

 

$\geq 3.\sqrt[6]{\frac{(a+b)^2(b+c)^2(c+a)^2}{8(a+b^2)(b+c)^2(c+a)^2}}=\frac{3\sqrt{2}}{2}$ 

Xảy ra dấu $=$ khi $a=b=c=\frac{1}{\sqrt{3}}$




#566805 Topic tổng hợp một số bất đẳng thức trong kì thi MO các nước

Đã gửi bởi Nguyen Minh Hai on 19-06-2015 - 09:22 trong Bất đẳng thức - Cực trị

 

Bài 117 (CĐTMO 2001) : Xét các số thực dương $a,b,c$ thỏa mãn điều kiện 

                                                                          $21ab+2bc+8ca\leq 12$
                                        Tìm giá trị nhỏ nhất của biểu thức $P=\frac{1}{a}+\frac{2}{b}+\frac{3}{c}.$ 

 

Spoiler


 

Bài toán này đã có khá nhiều cách giải trên diễn đàn

http://diendantoanho...-frac2b-frac3c/

http://diendantoanho...c3cgeq-frac152/

Thêm một cách mà mình vừa nghĩ ra  :icon10: 

Từ điểm rơi của bài toán là $(a,b,c)=\left ( \frac{1}{3};\frac{4}{5};\frac{3}{2} \right )$

$\Rightarrow 36a=15b=8c=12$

Do đó ta đặt: $\left\{\begin{matrix} 36a=x & & & \\ 15b=y & & & \\ 8c=z & & & \end{matrix}\right.$

Bài toán trở thành: 

Cho $x,y,z>0$ thõa mãn: $\frac{7xy}{180}+\frac{yz}{60}+\frac{zx}{36} \leq 12$.TÌm GTNN của:

$P=\frac{36}{x}+\frac{30}{y}+\frac{24}{z}$

Từ giả thiết ta có: 

$2160 \geq 7xy+3yz+5zx \geq 15\sqrt[15]{x^7y^7.y^3z^3.z^5x^5}$

$\Rightarrow x^6y^5z^4 \leq 12^5$            (Theo $AM-GM$)

Do đó: $P=6(\frac{6}{x}+\frac{5}{y}+\frac{4}{z})\geq 6.15\sqrt[15]{\frac{1}{x^6y^5z^4}} \geq 90.\sqrt[15]{\frac{1}{12^{15}}}=\frac{15}{2}$

$\Rightarrow GTNN_{P}=\frac{15}{2}$ khi $(a,b,c)=\left ( \frac{1}{3};\frac{4}{5};\frac{3}{2} \right )$

[spoiler] Đề thi vòng 16 Vio 9 năm nay cũng ra bài này [\spoiler]




#523981 TOPIC VỀ CÁC BÀI HÌNH HỌC LỚP 7,8

Đã gửi bởi Nguyen Minh Hai on 11-09-2014 - 22:35 trong Hình học

Bài 1: Cho hình thoi ABCD có $\widehat{BAD}=120^{\circ}$ , điểm M thuộc cạnh AB , DM cắt BC tại N, CM cắt AN tại E. Chứng minh rằng:

            a) $\Delta AMD \sim \Delta CDN$

            b) $\Delta AME \sim \Delta CMB$

Bài 2: Cho $\Delta ABC$ có $AB > AC$

            a) Vẽ đường cao BM, CN của tam giác. CMR:

                   $\Delta ABM \sim \Delta ACN$    và       $\widehat{AMN}=\widehat{ABC}$

            b) Lấy điểm K thuộc AB sao cho BK = AC. E là trung điểm BC, F là trung điểm AK. CMR:

                  EF // tia phân giác Ax của $\widehat{BAC}$




#554432 Violympic 2015

Đã gửi bởi Nguyen Minh Hai on 16-04-2015 - 19:29 trong Góc giao lưu

19/5 mới có kết quả chính xác .
http://violympic.vn/...il.aspx?ID=1274

:( 




#552926 Violympic 2015

Đã gửi bởi Nguyen Minh Hai on 10-04-2015 - 15:12 trong Góc giao lưu

em 230 :( 




#554491 Violympic 2015

Đã gửi bởi Nguyen Minh Hai on 16-04-2015 - 21:57 trong Góc giao lưu

cái thông báo đó là của năm ngoái, đăng ngày 28/4/2014, lịch mở cũng ghi rõ là 19/5/2014. Lần sau bạn nên đọc kỹ hơn!

À há....nhầm @@ 




#588873 Đăng ký tham gia dự thi VMEO IV

Đã gửi bởi Nguyen Minh Hai on 14-09-2015 - 15:30 trong Thông báo chung

Họ tên: Nguyễn Minh Hải

Nick trong diễn đàn (nếu có): Nguyen Minh Hai

Năm sinh: 2000

Hòm thư: [email protected]

Dự thi cấp: THCS , THPT 




#574757 Ôn kỉ niệm rồi giao lưu chút

Đã gửi bởi Nguyen Minh Hai on 23-07-2015 - 00:25 trong Góc giao lưu

À, anh còn 2,3 tải sách toán chuyên kìa. Đứa nào nịnh anh anh cho >:)  Bởi cũng không dùng nữa =))

Anh Giang đẹp trai hào phòng quá nhỉ! Lại còn hi sinh vì sự phát triển của Diễn Đàn nói riêng cũng như của nền Toán Học Việt Nam nói chung nữa!

Anh cho em xin sách!  >:)  >:) 

Spoiler




#573888 Ôn kỉ niệm rồi giao lưu chút

Đã gửi bởi Nguyen Minh Hai on 18-07-2015 - 22:42 trong Góc giao lưu

 Chú mày chuẩn bị yêu xa cmnr >:)

 Yên tâm là ở nhà Huyền đã có anh chăm sóc >:)  >:)  >:)  >:)  >:)  >:)  >:)  >:)

À anh cảm ơn...À mà chú nuôi dùm thằng nhỏ luôn nhé!  >:)  >:)  >:)  >:)  >:)  >:)




#573826 Ôn kỉ niệm rồi giao lưu chút

Đã gửi bởi Nguyen Minh Hai on 18-07-2015 - 20:28 trong Góc giao lưu

Mấy chú nhớ nuôi thằng nhỏ giùm anh luôn nhé! Anh cảm ơn!  >:)  >:)  >:)  >:)  >:)  >:)  >:)  >:)




#573777 Ôn kỉ niệm rồi giao lưu chút

Đã gửi bởi Nguyen Minh Hai on 18-07-2015 - 17:10 trong Góc giao lưu

Trai VMF cũng đào hoa ghê gớm nhỉ  >:)  >:)  >:)  >:)  >:)  >:)  >:)

 

Spoiler
 




#640894 VMF's Marathon Bất Đẳng Thức Olympic

Đã gửi bởi Nguyen Minh Hai on 17-06-2016 - 17:32 trong Bất đẳng thức và cực trị

Vế phải là mũ $3$ mới phải  :mellow:

Mình nhầm :3 

 

P/s: Long Vá xóa bài giùm anh  :wacko:




#640882 VMF's Marathon Bất Đẳng Thức Olympic

Đã gửi bởi Nguyen Minh Hai on 17-06-2016 - 16:24 trong Bất đẳng thức và cực trị

Bài 35:(VQBC) Cho $a,b,c$ là các số thực ko âm, chứng minh rằng : 

$$2(\frac{a}b{+\frac{b}c{+\frac{c}{a}}})+1\geq \frac{21(a^{2}+b^{2}+c^{2})}{(a+b+c)^{2}}$$

 

Lời giải bài 35. (Cách khác)

 

Sử dụng bổ đề 

$$(x+y+z)^3 \geqslant \frac{27}{4}(a^2b+b^2c+c^2a+abc)$$

ta có

$$left(\frac{a}{b}+\frac{b}{c}+\frac{c}{a} \right )^3 \geqslant \frac{27}{4}\left( \frac{a^3+b^3+c^3}{abc}+1 \right)$$

 

Do đó BĐT cần chứng minh trở thành 

 

$$54\left( \frac{a^3+b^3+c^3}{abc}+1 \right) \geqslant \left( \frac{21(a^2+b^2+c^2)}{(a+b+c)^2}-1\right)^2$$

 

$$\Leftrightarrow 54\left( \frac{a^3+b^3+c^3}{abc}-3 \right) \geqslant \left( \frac{441(a^2+b^2+c^2)^2}{(a+b+c)^4}-49 \right)-\left(\frac{42(a^2+b^2+c^2}{(a+b+c)^2}-14\right)$$

 

$$\Leftrightarrow \frac{54(a+b+c)(\sum a^2-\sum ab)}{abc} \geqslant \frac{14(\sum a^2-\sum ab)\left[21\sum a^2+7(a+b+c)^2\right]}{(a+b+c)^4}-\frac{28(\sum a^2-\sum ab)}{(a+b+c)^2}$$

 

$$\Leftghtarrow \left(\sum a^2-\sum ab \right)\left( \frac{27(a+b+c)}{abc}+\frac{14}{(a+b+c)^2}-\frac{147\sum a^2+49(a+b+c)^2}{(a+b+c)^4} \right)$$

 

Do đó ta chỉ cần chứng minh 

 

$$\frac{27(a+b+c)}{abc}+\frac{14}{(a+b+c)^2}-\frac{147\sum a^2+49(a+b+c)^2}{(a+b+c)^4} \geqslant 0$$

 

Chuẩn hóa $a+b+c=1$ thì ta cần chứng minh 

$$\frac{27}{abc}+14 \geqslant 147(a^2+b^2+c^2)+49$$

$$\Leftrightarrow 147abc(a^2+b^2+c^2)+35abc \leqslant 27$$

 

BĐT cuối luôn đúng do 

$$abc \leqslant \frac{1}{27}$$

và 

$$a^2+b^2+c^2 \leqslant (a+b+c)^2 =1$$

 

BĐT được chứng minh. Xảy ra đẳng thức khi $a=b=c$




#524374 $\frac{AB}{AE}+\frac{AD}{AF}=\frac{AC}{AO}$

Đã gửi bởi Nguyen Minh Hai on 14-09-2014 - 09:49 trong Hình học

Các anh các chị giải giùm em bài này với ạ!

Cho $\Delta ABC$ , trên BC lấy M, N sao cho $\widehat{BAM}=\widehat{CAN}$

CMR:  a) $\frac{BM}{CN}.\frac{CM}{BN}=(\frac{AM}{AN})^2$

             b) $\frac{BM}{CN}.\frac{BN}{CM}=(\frac{AB}{AC})^2$

             c) $\frac{BM}{CN}+\frac{CM}{BN}\geq 2.\frac{AM}{AN}$




#639955 VMF's Marathon Bất Đẳng Thức Olympic

Đã gửi bởi Nguyen Minh Hai on 13-06-2016 - 00:20 trong Bất đẳng thức và cực trị

 

Bài toán 31 (Võ Quốc Bá Cẩn). Cho các số thực dương $a,b,c$ thỏa mãn $a+b+c=5$. Chứng minh
$$a^2b+c^2a+2abc \leq 20.$$

 

Lời giải bài 31. Đặt $f(a,b,c)=a^2b+c^2a+2abc$

Ta xét $2$ trường hợp.

Nếu $a \geqslant c$, ta sẽ chứng minh 

$$f(a,b,c) \leqslant f(a,b,a)$$

$$\Leftrightarrow (a-c)(a+c+2b) \geqslant 0$$

 

Nếu $a \leqslant c$, ta sẽ chứng minh 

$$f(a,b,c) \leqslant f(c,b,c)$$

$$\Leftrightarrow b(c^2-a^2)+c^2(c-a)+2bc(c-a) \geqslant 0$$

 

Do đó ta chỉ cần chứng minh BĐT trong trường hợp $a=c=\frac{5-b}{2}$

$$\Leftrightarrow \left ( \frac{5-b}{2}\right )^2b+\left ( \frac{5-b}{2} \right)^3+2b\left (\frac{5-b}{2} \right )^2 \leqslant 20$$

$$\Leftrightarrow  b^3-9b^2+15b-7 \leqslant 0$$

$$\Leftrightarrow (b-7)(b-1)^{2} \leqslant 0$$

BĐT cuối luôn đúng do $b<5$

Bài toán được chứng minh. Xảy ra đẳng thức khi $a=c=2,b=1$

 

Bài toán 32.  (Vasile Cirtoaje) Cho các số thực dương $a,b,c,d$. Chứng minh rằng 

$$2(a^3+1)(b^3+1)(c^3+1)(d^3+1) \geqslant (1+abcd)(1+a^2)(1+b^2)(1+c^2)(1+d^2)$$